LSAT and Law School Admissions Forum

Get expert LSAT preparation and law school admissions advice from PowerScore Test Preparation.

 Administrator
PowerScore Staff
  • PowerScore Staff
  • Posts: 8919
  • Joined: Feb 02, 2011
|
#24029
Complete Question Explanation

Weaken. The correct answer choice is (A)

In this Stimulus, Dr. Ruiz concludes that Dr. Smith should not be included on a panel that examines the dangers of second-hand cigarette smoke. One of the reasons for this is because Dr. Ruiz, as organizer of the panel, wants the panel to examine the issue objectively before coming to any conclusion. Dr. Smith has expressed some strong antismoking views in public and is qualified to serve on this panel.

Again, in attacking the stimulus, why does Dr. Ruiz reach this conclusion? He states that he wants the panel to examine the issue in an unbiased manner before reaching its findings, and apparently he fears Dr. Smith’s outspoken views will compromise the panel’s objectivity.

The Question Stem reveals that this is a weaken question. In typical LSAT fashion however, it is not asked directly; instead, the question is phrased thusly: “which of the following, if true, provides the strongest basis for countering Dr. Ruiz’s argument that Dr. Smith should not be included on the panel?” Do not let this wordiness distract you. Instead, pause and examine the question. If you want to counter Dr. Ruiz’s argument, you will dispute his conclusion that Dr. Smith should be omitted from the panel, and this question asks what is the strongest way to do this. In other words, what is the best way to weaken Dr. Ruiz’s conclusion that Dr. Smith should not be included. In a speeded test as this one, the test-makers are counting on there being a few errors made because of careless reading; that is one of the main reasons why questions are asked in convoluted manners. If you are not careful in reading the Question Stem, you may approach this question as a strengthen question rather than a weaken question.

Answer Choice (A): This is the correct answer choice. It gives an example of how objectivity could be preserved while still including people who have strong opinions: include people with conflicting views. The presence of conflicting views gives the appearance of objectivity. Because this answer weakens Dr. Ruiz’s conclusion the most, it is the correct answer choice.

Answer Choice (B): This answer strengthens Dr. Ruiz’s argument. His primary concern is lack of objectivity, and if new evidence is not accepted unless it supports someone’s view, his concerns are well-founded, not weakened.

Answer Choice (C): This answer does not strengthen Dr. Ruiz’s argument but it also does not weaken it. Answer Choice (C) only deals with the likelihood of lively discussions, not the lack of objectivity, which is the main premise for Dr. Ruiz’s conclusion.

Answer Choice (D): This answer discusses fundraising, not objectivity. While, Answer Choice (D) could weaken the conclusion that Dr. Smith should be excluded, it introduces something entirely new to the equation – fundraising. Having already seen Answer Choice (A) as a good answer, Answer Choice (D) should be eliminated. It is an answer that could be right, but it certainly is not more correct that Answer Choice (A).

Answer Choice (E): This answer strengthens Dr. Ruiz’s conclusion and therefore should be eliminated.
 jrc3813
  • Posts: 53
  • Joined: Apr 16, 2017
|
#36948
I have a question regarding A. The stimulus never talks about the other members of the panel so we don't know that if she was added to the panel there would be people of conflicting views. It's possible that she would be the only one with strong views and the benefit to her being added would be unclear. If Ruiz was talking about not adding doctors in general who have outspoken views, A would be the perfect counter. But since it only mentions one doctor being considered it's a bit weaker. It's still the best answer but would I be correct to say that A isn't perfect in this respect?
 Luke Haqq
PowerScore Staff
  • PowerScore Staff
  • Posts: 722
  • Joined: Apr 26, 2012
|
#37437
Hi jrc3813,

As I read it, (A) clearly appears to weaken Dr. Ruiz's argument. The fact that the stimulus doesn't talk about other members isn't necessarily relevant; the question asks us to suppose that the answer choices are true.

Dr. Ruiz's argument is that he wants a panel that proceeds in an "unbiased manner." If (A) were true--even though it is talking about a group larger than just Dr. Smith--then it would suggest why Dr. Ruiz would be wrong to exclude Dr. Smith because of her "outspoken antismoking views in public." In other words, Dr. Ruiz's argument seems to be that Dr. Smith is not unbiased regarding secondhand smoke danger--so therefore she doesn't belong on a panel on secondhand smoke danger, if the purpose is to have that panel be unbiased.

If (A) were true--if people with "strong but conflicting views on a particular topic" are "more likely to reach an unbiased conclusion," then the fact that Dr. Smith "has expressed outspoken antismoking views in public" (i.e., she has strong views on a particular topic) would suggest her inclusion on a panel would help support Dr. Ruiz's goal of an unbiased panel. That is, it "provides the strongest basis for countering Dr. Ruiz' argument that Dr. Smith should not be included on the panel."

Hope that helps!
 caroline222
  • Posts: 18
  • Joined: Jan 07, 2021
|
#83822
Hi Powerscore, I don't really understand how A weakens. I understand why the previous instructor said that we have to accept the answer choices as true in weaken questions, but even if we accept this as true, I don't see how it weakens the conclusion that Dr. Smith cannot be included on the panel. We don't know if anyone on the panel will have conflicting views, so how does this give us any reason to believe that the conclusion doesn't follow from the premises? Wouldn't we need to know if there were people who had views that conflicted with Dr. Smiths' in order for this answer choice to work?
 Adam Tyson
PowerScore Staff
  • PowerScore Staff
  • Posts: 5153
  • Joined: Apr 14, 2011
|
#83928
You're putting too much pressure on the answers here, caroline222. The correct answer doesn't need to prove that the author is incorrect, or support the opposing position that Dr. Smith should be on the panel. All it has to do is raise some doubts about the conclusion. Focus on the relationship between the premises and the conclusion.

Simply put, Dr. Ruiz says "Dr. Smith is biased and the panel should be unbiased, so Smith cannot be on the panel."

Answer A weakens that argument by suggesting that an unbiased panel can be comprised of biased members. Smith's bias, by itself, is no longer a good reason to keep her off the panel!

Also consider that Dr. Ruiz made a "whole to part" flaw here, in that she thinks an unbiased panel (the whole) must be comprised only of unbiased members (the parts). Answer A points out that flaw. That's enough to weaken the argument, even if it doesn't completely prove Dr. Ruiz is wrong in her conclusion (as there may be other reasons to keep Dr. Smith off the panel, or the rest of the panel may be made up of only unbiased members).

Weaken just means raise doubt! Don't require any more of the answer than just that.
User avatar
 ashpine17
  • Posts: 321
  • Joined: Apr 06, 2021
|
#101859
I liked A initially but ultimately went for C after weighing both answers :( I liked both A and C because both contained something about a qualified person who had strong public views but I didn't like A because some of the wording seemed to be off from what was in the stimulus. I didn't like that it had conflicting and it said unbiased conclusion versus unbiased manner. why are these not issues? As for C, I didn't like lively discussion because I didn't think it was directly relevant to being unbiased but I thought it was wishy-washy enough to go either way, why is this wrong?
 Rachael Wilkenfeld
PowerScore Staff
  • PowerScore Staff
  • Posts: 1358
  • Joined: Dec 15, 2011
|
#101865
Hi ash,

One issue with answer choice (c) is the result that answer choice (c) suggests. That answer choice states that including Dr. Smith would make it more likely that the panel has lively discussions. But the goal of the panel isn't to have lively discussions---it's to come up with unbiased conclusion.

One of your concerns here seems to be that answer choice (a) has information outside of the information in the stimulus. That's ok in this question type. In fact, we expect it. We are trying to find what other information would weaken our conclusion. Typically that means that the answer choices may include information from outside the argument that is relevant to the argument. Answer choice (a) is extremely relevant. It states that including qualified passionate people with conflicting views is more likely to reach an unbiased result than a panel made up of people with only private views. Dr. Ruiz is looking for a panel that appears unbiased---one that doesn't include people with strong public views on the topic. But by excluding those with biased views, Dr. Ruiz's panel will be less likely to reach an unbiased result.

Hope that helps!

Get the most out of your LSAT Prep Plus subscription.

Analyze and track your performance with our Testing and Analytics Package.